抱歉,您的浏览器无法访问本站
本页面需要浏览器支持(启用)JavaScript
了解详情 >

级数的概念与性质

定义 设数列 {un}\{u_n\} ,把和式 u1+u2++un+u_1+u_2+\cdots+u_n+\cdots 称为无穷级数,简称 级数。其中 unu_n 称为级数 n=1un\displaystyle \sum_{n=1}^\infin u_n通项或者一般项,把 Sn=k=nuk\displaystyle S_n=\sum_{k=n}^\infin u_k 称为级数的部分和或者n\boldsymbol n 项和

定义:级数收敛 如果极限 limnSn\displaystyle\lim_{n \to \infin } S_n 存在,则称级数 n=1un\displaystyle \sum_{n=1}^\infin u_n 收敛,极限值 SS 称为级数的和。记作 S=n=1un\displaystyle S=\sum_{n=1}^\infin u_n。如果极限不存在,称为发散。当级数 n=1un\displaystyle \sum_{n=1}^\infin u_n 收敛与 SS,把 rn=k=n+1uk=SSn\displaystyle r_n=\sum_{k=n+1}^\infin u_k=S-S_n 称为级数的余和,此时 limnrn=0\displaystyle \lim_{n \to \infin} r_n=0

image-20230510100930413

![image-20230510102113235](C:\Users\Steven Meng\AppData\Roaming\Typora\typora-user-images\image-20230510102113235.png)


判断调和级数 n=11n\displaystyle \sum_{n=1}^\infin \frac{1}{n} 的敛散性。

利用 ln(1+x)<x\ln(1+x)<x,得到 1n>ln(1+1n)\frac{1}{n}>\ln\left(1+\frac{1}{n}\right)

得到发散。


判别级数 n=1arctan12n2\displaystyle \sum_{n=1}^\infin\arctan \frac{1}{2n^2} 的敛散性。

需要使用

arctanx2x11+x2x1=arctanx2arctanx1\arctan \frac{x_2-x_1}{1+x_2\cdot x_1}=\arctan x_2-\arctan x_1

进行裂项。因此 un=arctan(2n+1)arctan(2n1)u_n=\arctan(2n+1)-\arctan(2n-1)

limnSn=limnarctan(2n+1)arctan1=π4\lim_{n\to \infin} S_n=\lim_{n\to\infin} \arctan(2n+1)-\arctan 1=\frac{\pi}{4}

另外的方法:使用 n=1arctan12n2\displaystyle \sum_{n=1}^\infin\arctan \frac{1}{2n^2}n=112n2\displaystyle \sum_{n=1}^\infin\frac{1}{2n^2} 有相同的敛散性。


判定级数 a11+a1+a2(1+a1)(1+a2)++an(1+a1)(1+a2)(1+an)+\displaystyle \frac{a_1}{1+a_1}+\frac{a_2}{(1+a_1)(1+a_2)}+\cdots+\frac{a_n}{(1+a_1)(1+a_2)\cdots(1+a_n)}+\cdots

可以进行列项,

an(1+a1)(1+a2)(1+an)=1(1+a1)(1+a2)(1+an1)1(1+a1)(1+a2)(1+an)\frac{a_n}{(1+a_1)(1+a_2)\cdots(1+a_n)}=\frac{1}{(1+a_1)(1+a_2)\cdots(1+a_{n-1})}-\frac{1}{(1+a_1)(1+a_2)\cdots(1+a_n)}

得到

Sn=11(1+a1)(1+a2)(1+an)S_n=1-\frac{1}{(1+a_1)(1+a_2)\cdots(1+a_n)}

由于 ak>0a_k>0,所以 SnS_n 单调递增,而且 Sn<1S_n<1,因此极限单调有界,有极限。

性质

  1. 级数的每一项乘以一个不为零的常数 cc,得到的级数和原级数有相同的敛散性。

  2. 如果 n=1un,n=1vn\sum_{n=1}^\infin u_n,\sum_{n=1}^\infin v_n 收敛,则级数 n=1(un+vn)\sum_{n=1}^\infin (u_n+v_n) 也收敛,而且 n=1(un+vn)=n=1un+n=1vn\sum_{n=1}^\infin (u_n+v_n)=\sum_{n=1}^\infin u_n+\sum_{n=1}^\infin v_n,但是反向结论不成立。

  3. 将级数添加、去掉或者改变有限项,不改变极限的收敛性,但是可能改变级数收敛到的值。如果讨论级数的敛散性,直接写 un\sum u_n,因为下界无所谓。

  4. 收敛级数任意加括号组成的新级数仍然收敛,反之不然,例如 (11)\sum (1-1). 如果加括号之后组成的级数发散,则原级数一定发散。

    利用这个结论说明 1n\sum \frac{1}{n} 发散,加括号 1+12+(13+14)++(12n1+1+12n1+2++12n)1+\frac{1}{2}+(\frac{1}{3}+\frac{1}{4})+\cdots+(\frac{1}{2^{n-1}+1}+\frac{1}{2^{n-1}+2}+\cdots+\frac{1}{2^{n}})

  5. 级数收敛的必要条件:如果级数 n=1un\displaystyle \sum_{n=1}^\infin u_n 收敛,则 limnun=0\displaystyle \lim_{n\to \infin} u_n=0

    推论:若 limnun0\displaystyle \lim_{n\to \infin} u_n\not=0,则 n=1un\displaystyle \sum_{n=1}^\infin u_n 发散。

![image-20230510104323503](C:\Users\Steven Meng\AppData\Roaming\Typora\typora-user-images\image-20230510104323503.png)

​ 判断级数 n=1(1)n1(11n)n\sum_{n=1}^\infin (-1)^{n-1}(1-\frac{1}{n})^n 的敛散性。得到 limnan=1e0\lim_{n\to \infin} |a_n|=\frac{1}{e}\not=0。因此不收敛。

几何级数中的 qqnn 无关。

![image-20230510104623398](C:\Users\Steven Meng\AppData\Roaming\Typora\typora-user-images\image-20230510104623398.png)

正项级数及其敛散性判别法

常数项级数unu_n 为常数时,级数 n=1un\sum_{n=1}^\infin u_n 称为常数项级数。

正项级数an>0a_n >0 时,级数 n=1an\sum_{n=1}^\infin a_n 称为正项级数。正项级数的部分和单调递增。

收敛原理 正项级数 n=1an\sum_{n=1}^\infin a_n 收敛的充分必要条件是部分和数列 {Sn}\{S_n\} 有上界。(单增有上界的数列必然收敛)

有限项负项数列 若级数 n=1an\sum_{n=1}^\infin a_n 满足 N>0\exists N>0,使得当 n>Nn>N 时,有 an0a_n \ge 0,那么级数 n=1an\sum_{n=1}^\infin a_n 收敛的充要条件是 n=1an\sum_{n=1}^\infin a_n 的部分和数列 {Sn}\{S_n\} 有上界。

比较判别法

定理1,通过比较相对大小

如果正项级数 n=1an,n=1bn\displaystyle\sum_{n=1}^\infin a_n,\sum_{n=1}^\infin b_n,满足条件 0<ancbn(n=1,2,)0<a_n \le {\color{blue} c}\cdot b_n(n=1,2,\cdots),其中 c>0c>0,则

  1. n=1an\displaystyle\sum_{n=1}^\infin a_n 发散,级数 n=1bn\displaystyle\sum_{n=1}^\infin b_n 发散。
  2. n=1bn\displaystyle\sum_{n=1}^\infin b_n 收敛,级数 n=1an\displaystyle\sum_{n=1}^\infin a_n 收敛。

推论:an+1anbn+1bn\displaystyle \frac{a_{n+1}}{a_n} \le \frac{b_{n+1}}{b_n} 也可以推出条件。

![image-20230510111015380](C:\Users\Steven Meng\AppData\Roaming\Typora\typora-user-images\image-20230510111015380.png)

结论: pp-级数 n=11np\sum_{n=1}^\infin \frac{1}{n^p}p>1p>1 时收敛,当 p1p\le 1 时发散。

可以通过广义积分是否趋于某个常数判断。

用积分得到不等式

![image-20230510111658814](C:\Users\Steven Meng\AppData\Roaming\Typora\typora-user-images\image-20230510111658814.png)

利用

limsin1n1n=1\lim \frac{\sin \frac{1}{n}}{\frac{1}{n}}=1

ε>0,N\forall \varepsilon >0,\exists N,当 n>Nn>N 时,

sin1n1n<ε\left|\frac{\sin \frac{1}{n}}{\frac{1}{n}}\right| <\varepsilon

因此

1n(1ε)<sin1n<1n(1+ε)\frac{1}{n}(1-\varepsilon) < \sin \frac{1}{n} < \frac{1}{n}(1+\varepsilon)

对于 ε0=12\varepsilon_0=\frac{1}{2},得到

sin1n>12n\sin\frac{1}{n}>\frac{1}{2n}

由于 n=112n\sum_{n=1}^\infin \frac{1}{2n} 发散,所以 n=1sin1n\sum_{n=1}^\infin \sin \frac{1}{n} 发散。


利用不等式 12nan<(2n1)!!(2n)!!bn<12n+1cn\displaystyle \underbrace{\frac{1}{2\sqrt{n}}}_{a_n}< \underbrace{\frac{(2n-1)!!}{(2n)!!}}_{b_n}<\underbrace{\frac{1}{\sqrt{2n+1}}}_{c_n} 证明级数 n=1(2n1)!!(2n)!!\displaystyle \sum_{n=1}^\infin \frac{(2n-1)!!}{(2n)!!} 发散,而级数 n=1(2n3)!!(2n)!!\displaystyle \sum_{n=1}^\infin \frac{(2n-3)!!}{(2n)!!} 收敛。

不等式可以使用比较相邻两项的比值推出:

an/an1=11n,bn/bn1=2n12na_n/a_{n-1}=\sqrt{1-\frac{1}{n}},b_n/b_{n-1}=\frac{2n-1}{2n}

泰勒展开,可以推出左小于右。

bn/bn1=2n12n,cn/cn1=122n+1b_n/b_{n-1}=\frac{2n-1}{2n},c_n/c_{n-1}=\sqrt{1-\frac{2}{2n+1}}

可得左小于右。

bn>anbnbn/(2n1)<cn/(2n1)bn/(2n1)b_n>a_n \Rightarrow \sum b_n 发散\\ b_n/(2n-1)<c_n/(2n-1) \Rightarrow \sum b_n/(2n-1) 收敛

比较判别法的极限形式

正项级数 n=1an,n=1bn\sum_{n=1}^\infin a_n,\sum_{n=1}^\infin b_n,满足 limnbnan=l\lim_{n\to \infin} \frac{b_n}{a_n}=l,则

  1. 0<l<+0<l<+\infin 时,级数 n=1bn\sum_{n=1}^\infin b_nn=1an\sum_{n=1}^\infin a_n 敛散性相同。
  2. l=0l=0 时,若级数 n=1an\sum_{n=1}^\infin a_n 收敛,则 n=1bn\sum_{n=1}^\infin b_n 也收敛。
  3. l=+l=+\infin 时,若级数 n=1an\sum_{n=1}^\infin a_n 发散,则 n=1bn\sum_{n=1}^\infin b_n 也发散。

证明: ε>0,NZ+\forall \varepsilon >0,\exists N \in \Z^+,当 n>Nn>N 时,(lε)an<bn<(1+ε)an(l-\varepsilon)a_n <b_n <(1+\varepsilon) a_n

0<l<+0<l<+\infin 时,取 ε0>0\varepsilon_0>0,使得 lε0>0l-\varepsilon_0>0,由比较判别法得到两个级数敛散性相同。

![image-20230510112901273](C:\Users\Steven Meng\AppData\Roaming\Typora\typora-user-images\image-20230510112901273.png)

![image-20230510113147312](C:\Users\Steven Meng\AppData\Roaming\Typora\typora-user-images\image-20230510113147312.png)

![image-20230510113349569](C:\Users\Steven Meng\AppData\Roaming\Typora\typora-user-images\image-20230510113349569.png)

一般是和 pp-级数比较,称为 pp-判别法。


这个结论只能用于正项级数,反例是,考虑 un=(1)nn,vn=(1)nn+1+(1)n\displaystyle u_n=\frac{(-1)^n}{\sqrt{n}},v_n=\frac{(-1)^n}{\sqrt{n+1}+(-1)^n}un/vn1u_n /v_n \to 1,但是 vnv_n 发散。

limnanbn=0\displaystyle \lim_{n \to \infin} \frac{a_n}{b_n}=0 时,n=1bn\displaystyle \sum_{n=1}^\infin b_n 收敛 ⇏\not\Rightarrow n=1an\displaystyle \sum_{n=1}^\infin a_n 收敛。

pp-判别法

n=1an\sum_{n=1}^\infin a_n 是正项级数,且 limnnpan=l\lim_{n\to\infin} n^p \cdot a_n=l,则

  1. 0l<+0\le l<+\infinp>1p>1 时,级数 n=1an\sum_{n=1}^\infin a_n 收敛;
  2. 0<l+0<l\le +\infinp1p\le 1 时,级数 n=1an\sum_{n=1}^\infin a_n 发散。

类比

limx+xpf(x)=l\lim_{x\to+\infty}x^pf(x)=l,则 f(x)f(x)1/xp1/x^p 同阶:

  1. p>1p>10l<+0\le l <+\infty 时,a+f(x)dx\int_a^{+\infty}f(x)\mathrm d x 收敛,是因为此时 1/xp1/x^p 是收敛的。
  2. p1p\le10<l+0<l\le+\infty 时,a+f(x)dx\int_a^{+\infty}f(x)\mathrm d x 发散,是因为此时 1/xp1/x^p 是发散的。

关键是求出 ana_nnn 的几阶无穷小,当然如果是 ln(n)\ln (n) 之类的,可以取 1/n1/n 比较,例如判断 n=11ln2(n+1)\displaystyle \sum_{n=1}^\infin \frac{1}{\ln^2(n+1)} 的敛散性。

bn=1nb_n=\frac{1}{n},得到

limnanbn=\lim_{n \to \infin} \frac{a_n}{b_n}=\infin

而且 bn\sum b_n 本身不收敛,因此 ana_n 不收敛。

![image-20230510113948447](C:\Users\Steven Meng\AppData\Roaming\Typora\typora-user-images\image-20230510113948447.png)


证明两个级数一个收敛一个发散,求和是发散的。

image-20230510210414980


用比较判别法或比较判别法的极限形式判别下列级数的敛散性:

n=1an1+a2n(a>0)\sum_{n=1}^\infin \frac{a^n}{1+a^{2n}} (a>0)

先进行变形:

1an+an\frac{1}{a^{-n}+a^n}

然后分类讨论

  1. a=1a=1,发散。
  2. 0<a<10<a<1,得到 an0a^n \to 0,因此原式 <1/an<1/a^{-n},收敛。
  3. a>1a>1,得到 an0a^{-n}\to0,因此原式 <1/an<1/a^n,收敛。

设正项级数 n=1an\displaystyle \sum_{n=1}^\infin a_n 收敛,且 an+1an(n=1,2,)a_{n+1} \le a_n(n=1,2,\cdots)

证明

  1. 级数 n=1n(anan+1)\displaystyle \sum_{n=1}^\infin n(a_n-a_{n+1}) 收敛。

    首先,观察到这个式子是正项级数,把这个式子展开,得到 k=1naknan+1k=1nak\displaystyle \sum_{k=1}^n a_k-na_{n+1}\le \sum_{k=1}^n a_k,因此 n=1n(anan+1)\sum_{n=1}^\infin n(a_n-a_{n+1}) 收敛。注意必须满足正项级数的条件,如果不是正项,反复横跳也可以发散。

  2. limnnan=0\displaystyle \lim_{n \to \infin } n a_n=0

    limnnan+1\lim_{n \to \infin} na_{n+1} 存在,得到 limnnan\lim_{n \to \infin} na_n 存在?

    得到

    limnan1n=l\lim_{n \to \infin} \frac{a_n}{\frac{1}{n}}=l

    1/n1/n 发散,ana_n 收敛得到 l=0l=0

比值判别法和根值判别法

仅仅依赖级数本身结构来进行判别。

比值判别法

n=1an\displaystyle \sum_{n=1}^\infin a_n 为正项级数,且 limnan+1an=l\displaystyle\lim _{n \to \infin} \frac{a_{n+1}}{a_n}=l(或者 ++\infin),则:

  1. 0l<10\le l <1 时,级数 n=1an\displaystyle \sum_{n=1}^\infin a_n 收敛。
  2. 1<l+1<l\le +\infin 时,级数 n=1an\displaystyle \sum_{n=1}^\infin a_n 发散。
  3. l=1l=1 时,不能确定,需要特殊讨论。例如 an=1/n,an=1/n2a_n=1/n,a_n=1/n^2

a>0a>0,试着讨论级数 n=1ann!nn\displaystyle \sum_{n=1}^\infin \frac{a^n n!}{n^n} 的敛散性。

讨论

limnan+1(n+1)!(n+1)n+1ann!nn=limna(n+1)nn(n+1)n+1=limna(1+1n)n=ae\lim_{n \to \infin} \frac{\frac{a^{n+1}(n+1)!}{(n+1)^{n+1}}}{\frac{a^n n!}{n^n}}=\lim_{n \to \infin} \frac{a(n+1)n^n}{(n+1)^{n+1}}=\lim_{n\to \infin} \frac{a}{(1+\frac{1}{n})^n}=\frac{a}{e}

a>ea> e 时发散,a<ea<e 时收敛,那么 a=e\boldsymbol{a=e} 时如何,直觉是发散的,因为后面每个数近似一样。

正确证明是利用 (1+1n)n<e(1+\frac{1}{n})^n <e,得到 an+1>an>>a1=ea_{n+1}>a_n>\cdots>a_1=e


另外的方法是应用 Stirling 公式:n!2πn(ne)n,n\displaystyle n! \sim \sqrt{2\pi n} \left(\frac{n}{e}\right)^n,n \to \infin. a=ea=e 时,每一项近似 2πn\sqrt{2\pi n},因此发散。

根值判别法

n=1an\displaystyle \sum_{n=1}^\infin a_n 为正项级数,且 limnann=l\displaystyle \lim_{n \to \infin} \sqrt[n]{a_n}=l(或者 ++\infin

  1. 0l<10\le l<1 时,级数 n=1an\displaystyle \sum_{n=1}^\infin a_n 收敛;
  2. 1l+1\le l\le +\infin 时,级数 n=1an\displaystyle \sum_{n=1}^\infin a_n 发散。

因为后面每一项近似看成 an=lna_n=l^n,近似为一个等比数列。可以证明,如果满足比值条件,就可以满足根值条件。


试讨论 n=1n(a+1n)n\displaystyle \sum_{n=1}^\infin \frac{n}{\left(a+\frac{1}{n}\right)^n} 的敛散性。

得到

limnann=1a\lim_{n\to\infin} \sqrt[n]{a_n}=\frac{1}{a}

需要特别讨论 a=1a=1 的情况,由于

an=n(1+1n)na_n=\frac{n}{(1+\frac{1}{n})^n}\to \infin


判定级数 n=2nlnn(lnn)n\displaystyle \sum_{n=2}^\infin \frac{n^{\ln n}}{(\ln n)^n} 的敛散性。

limnann=limneln2n/nlnn=0<1\lim_{n\to\infin} \sqrt[n]{a_n}=\lim_{n \to \infin} \frac{e^{\ln ^2 n/n}}{\ln n}=0<1

所以收敛。


若级数 n=1anns(a>0,s>0)\displaystyle \sum_{n=1}^\infin \frac{a^n}{n^s}(a>0,s>0) 收敛,则 aass 的取值。

使用比值判别法必须讨论比值趋于 0 的情况。

limnan+1an=a(nn+1)sa\lim_{n \to \infin} \frac{a_{n+1}}{a_n}=a \left(\frac{n}{n+1}\right)^s \to a

0<a<10<a<1 时必然收敛。

a>1a>1 必然发散。

a=1a=1,代回原式得到 pp-级数,因此此时 s>1s>1.

积分判别法

n=1an\displaystyle \sum_{n=1}^\infin a_n 为正项级数,若非负函数 f(x)f(x)[1,+)[1,+\infin)单调减少,且对 nZ+\forall n \in \Z^+,有 an=f(n)a_n=f(n),则级数 n=1an\displaystyle \sum_{n=1}^\infin a_n 与反常积分 1f(x)dx\displaystyle \int_1^\infin f(x)\mathrm d x 有相同的敛散性。


讨论级数 n=21n(lnn)q\displaystyle \sum_{n=2}^\infin \frac{1}{n(\ln n)^q} 的敛散性,其中常数 q>0q>0

q1\boldsymbol{q\not=1}

2+dxx(lnx)q=(lnx)1q1q2+=limx+(lnx)1q1q(ln2)1q1q\int_2^{+\infin} \frac{\mathrm d x}{x(\ln x)^q}=\left.\frac{(\ln x)^{1-q}}{1-q}\right|_2^{+\infin}=\lim_{x \to +\infin} \frac{(\ln x)^{1-q}}{1-q}-\frac{(\ln 2)^{1-q}}{1-q}

0<q<10<q<1,得到前面趋于 ++\infin,发散。

q>1q>1,得到前面趋于 00,收敛于 (ln2)1qq1\displaystyle \frac{(\ln 2)^{1-q}}{q-1}

q=1\boldsymbol{q=1}

2+dxxlnx=ln(lnx)2+=+\int_2^{+\infin} \frac{\mathrm d x}{x\ln x}=\left.\ln(\ln x)\right|_2^{+\infin}=+\infin

发散。

注意:an=1nlnn\displaystyle a_n=\frac{1}{n\ln n}1n\dfrac{1}{n} 的高阶无穷小,但是 n=1an\displaystyle \sum_{n=1}^\infin a_n 发散。

![image-20230525120246524](C:\Users\Steven Meng\AppData\Roaming\Typora\typora-user-images\image-20230525120246524.png)

任意项级数敛散性的判别法

这里去掉一般项 an0(nZ+)a_n \ge 0(n \in \Z^+) 的限制。但是,在某些条件下还是有 0\ge 0 的条件。

交错项级数敛散性的判别法

各项正负交错,即形如 ±n=1(1)n1an(an>0)\displaystyle \pm \sum_{n=1}^\infin (-1)^{n-1} a_n (a_n >0) 的级数称为 交错级数

Leibniz 判别法

若交错级数 n=1(1)n1an\displaystyle \sum_{n=1}^\infin (-1)^{n-1} a_n 满足:

  1. 0<an+1an(nZ+)0<a_{n+1}\le a_n(n \in \Z^+)
  2. limnan=0\lim_{n \to \infin} a_n=0

则级数 n=1(1)n1an\displaystyle \sum_{n=1}^\infin (-1)^{n-1} a_n 收敛,且其余项级数满足 k=n+1(1)k1akan+1\left|\displaystyle \sum_{k=n+1}^\infin (-1)^{k-1} a_k\right| \le a_{n+1}

三点条件:an>0,an+1an,lim\boldsymbol{a_n>0,a_{n+1}\le a_n, \lim} 如果不满足 >0>0 的条件,可以全部取负号。


判别 i=1(1)n1(sin1n1n)\displaystyle \sum_{i=1}^\infin (-1)^{n-1} \left(\sin \frac{1}{n}-\frac{1}{n}\right) 的敛散性。

f(x)=sinxxf(x)=\sin x-x,则 f(x)=cosx10f'(x)=\cos x-1\ge 0,所以 an+1ana_{n+1} \le a_{n},而且 an0a_n \to 0,因此满足 Leibniz 判别法的条件,收敛。

![image-20230515091537979](C:\Users\Steven Meng\AppData\Roaming\Typora\typora-user-images\image-20230515091537979.png)


含有扰动项的处理方法,例如 n=1(1)n11n+(1)n1\displaystyle \sum_{n=1}^\infin (-1)^{n-1}\frac{1}{n+(-1)^{n-1}}

是不满足 Leibniz 判别法的条件的,这里我们分母凑成平方项:

n=1(1)n1n(1)n1n21=n=1(1)n1nn21n=11n21\sum_{n=1}^\infin (-1)^{n-1} \frac{n-(-1)^{n-1}}{n^2-1}=\sum_{n=1}^\infin (-1)^{n-1} \frac{n}{n^2-1} - \sum_{n=1}^\infin \frac{1}{n^2-1}

因此收敛。

Abel 判别法和 Dirichlet 判别法

若数列 {un}\{u_n\} 单调且有界,级数 n=1vn\displaystyle \sum_{n=1}^\infin v_n 收敛,则级数 n=1unvn\displaystyle \sum_{n=1}^\infin u_n v_n 收敛。

若数列 {un}\{u_n\} 单调且 limnun=0\lim_{n \to \infin} u_n =0,且级数 n=1vn\displaystyle \sum_{n=1}^\infin v_n 的部分和数列有界,则级数 n=1unvn\displaystyle \sum_{n=1}^\infin u_nv_n 收敛。

绝对收敛和条件收敛

n=1an\displaystyle \sum_{n=1}^\infin a_n 是任意项级数,若级数 n=1an\displaystyle \sum_{n=1}^\infin |a_n| 收敛,则称级数 n=1an\displaystyle \sum_{n=1}^\infin a_n 绝对收敛;若级数 n=1an\displaystyle \sum_{n=1}^\infin |a_n| 发散而 n=1an\displaystyle \sum_{n=1}^\infin a_n 收敛,则称级数 n=1an\displaystyle \sum_{n=1}^\infin a_n 条件收敛

定理 若级数 n=1an\displaystyle \sum_{n=1}^\infin a_n 绝对收敛,则 n=1an\displaystyle \sum_{n=1}^\infin a_n 必收敛。

证明:由于 0an+an2an0\le a_n + |a_n| \le 2 |a_n|,所以级数

n=1nan=n=1[(an+an)an]\sum_{n=1}^n a_n=\sum_{n=1}^\infin [(a_n+|a_n|)-|a_n|]

收敛。

定理 若级数 n=1an\displaystyle \sum_{n=1}^\infin a_n 绝对收敛,那么任意交换其各项的次序所得到的级数依然绝对收敛,且其和不变。反之如果级数只是收敛,交换次序得到的级数不一定收敛,比如交错级数

n=1(1)n11n\sum_{n=1}^\infin (-1)^{n-1}\frac{1}{\sqrt{n}}

定理 (Cauchy) 设级数 n=1an,n=1bn\displaystyle \sum_{n=1}^\infin a_n,\sum_{n=1}^\infin b_n 收敛于 S,TS,T,则 n=1m=1anbm\displaystyle \sum_{n=1}^\infin \sum_{m=1}^\infin a_n b_m 收敛于 STST(分配率)


判断 n=1sinnn2\displaystyle \sum_{n=1}^\infin \frac{\sin n}{n^2} 的敛散性。(相似 n=1(1)n1nsinπn\displaystyle \sum_{n=1}^\infin \frac{(-1)^{n-1}}{n}\sin\frac{\pi}{n}

n=1sinnn2n=11n2\sum_{n=1}^\infin \frac{|\sin n|}{n^2} \le \sum_{n=1}^\infin \frac{1}{n^2}

绝对收敛,因此收敛。


判断 n=1(1)n1ln(n+1)\displaystyle \sum_{n=1}^\infin (-1)^n \frac{1}{\ln (n+1)} 的敛散性。(相似 n=1(1)nlnnn\displaystyle \sum_{n=1}^\infin (-1)^n \frac{\ln n}{n}

可以用 Leibniz 判别法判别收敛,但是看看是否绝对收敛:

n=11ln(n+1)n=11n\displaystyle \sum_{n=1}^\infin \frac{1}{\ln (n+1)} \ge \sum_{n=1}^\infin \frac{1}{n}

因此 an\sum |a_n| 发散,绝对收敛不成立。这时 ana_n 条件收敛。因此,收敛不一定绝对收敛。


【例题13】证明级数 n=1(1)n(e1n11n)\displaystyle \sum_{n=1}^{\infty}(-1)^n\left(\mathrm{e}^{\frac{1}{\sqrt{n}}}-1-\frac{1}{\sqrt{n}}\right) 条件收敛。
证明: 设 un=e1n11n\displaystyle u_n=\mathrm{e}^{\frac{1}{\sqrt{n}}}-1-\frac{1}{\sqrt{n}}, 记 x=1n\displaystyle x=\frac{1}{\sqrt{n}},
x>0x>0 时, f(x)=ex1x>0f(x)=\mathrm{e}^x-1-x>0, 所以 un=f(1n)>0\displaystyle u_n=f\left(\frac{1}{\sqrt{n}}\right)>0,
limx0ex1xx2=12\displaystyle \lim _{x \rightarrow 0} \frac{\mathrm{e}^x-1-x}{x^2}=\frac{1}{2}
limne1n11n1n=12\displaystyle \lim _{n \rightarrow \infty} \frac{\mathrm{e}^{\frac{1}{\sqrt{n}}}-1-\frac{1}{\sqrt{n}}}{\frac{1}{n}}=\frac{1}{2},
因此级数 n=1(e1n11n)\displaystyle \sum_{n=1}^{\infty}\left(\mathrm{e}^{\frac{1}{\sqrt{n}}}-1-\frac{1}{\sqrt{n}}\right) 发散。

拉比判别法

对于任意级数 n=1an\sum_{n=1}^\infin a_n,如果存在 r>1,n0N+r>1,n_0\in \N^+,使得当 n>n0n>n_0 时,有 n(anan+11)r\displaystyle n\left(\left|\frac{a_n}{a_{n+1}}\right|-1\right)\ge r,那么级数 n=1an\displaystyle \sum_{n=1}^\infin a_n 绝对收敛。

an>0,an+1an>11n,nNa_n>0,\displaystyle \frac{a_{n+1}}{a_n}>1-\frac{1}{n},\forall n \in \N,则级数 n=1an\displaystyle \sum_{n=1}^\infin a_n 发散。

直接使用 an+1a2>1n\displaystyle \frac{a_{n+1}}{a_2} > \frac{1}{n},从而原级数发散。

重排、括号敛散性判断

重排

绝对收敛可以任意重排,保证敛散性不变,且收敛到相同的数。

条件收敛重排后,可以收敛到任意数或者到 ±\pm \infin.

括号

正项级数可以任意加括号,保证敛散性不变。

收敛级数可以任意加括号,保证敛散性不变。

函数项级数及其敛散性

设函数列 {un(x)} (n=1,2,)\{u_n(x)\} \ (n=1,2,\cdots) (每一项都是函数)在实数集合(一般为区间)X\boldsymbol X 上有定义,则和式 n=1un(x)\displaystyle \sum_{n=1}^\infin u_n(x) 称为 函数项级数。若数项级数 n=1un(x0) (x0X)\displaystyle \sum_{n=1}^\infin u_n(x_0)\ (x_0 \in \boldsymbol X) 收敛,则称 x0x_0 是级数 n=1un(x)\displaystyle \sum_{n=1}^\infin u_n (x)收敛点,否则称为 发散点。函数项级数 n=1un(x)\displaystyle \sum_{n=1}^\infin u_n(x) 的全体收敛点组成的集合 II 称为 收敛域

对于收敛域 II 中的每一点 xx,记 n=1un(x)\displaystyle \sum_{n=1}^\infin u_n(x) 的和为 S(x)S(x),即

S(x)=n=1un(x)(xI)S(x)=\sum_{n=1}^\infin u_n(x)\quad (x\in I)

称为 和函数

又记

Sn(x)=k=1nuk(x),rn(x)=k=n+1uk(x)S_n(x)=\sum_{k=1}^n u_k (x),r_n (x)=\sum_{k=n+1}^\infin u_k(x)

分别称为 部分和余和

![image-20230522143247555](C:\Users\Steven Meng\AppData\Roaming\Typora\typora-user-images\image-20230522143247555.png)

两个端点比较重要

![image-20230512201540694](C:\Users\Steven Meng\AppData\Roaming\Typora\typora-user-images\image-20230512201540694.png)

**【思考题】**求函数项级数 n=1xn1+x2n\displaystyle \sum_{n=1}^\infin \frac{x^n}{1+x^{2n}} 的收敛域。

根值判别法。发散点是 1,11,-1.

幂级数

其一般项函数都是幂函数。

幂级数及收敛半径

定义x0x_0 是一个定数,形如

n=0an(xx0)n=a0+a1(xx0)+a2(xx0)2+\sum_{n=0}^\infin a_n (x-x_0)^n =a_0+a_1(x-x_0)+a_2(x-x_0)^2+\cdots

的函数项级数称为 xx0x-x_0 的幂级数,简称 幂级数,其中 a0,a1,a2,a_0,a_1,a_2,\cdots 称为幂级数的系数。

若令 y=xx0y=x-x_0,则上述幂级数可以化为

n=0anyn=a0+a1y+a2y2+\sum_{n=0}^\infin a_n y^n =a_0+a_1y+a_2 y^2+\cdots

因此只需要讨论 n=0anyn\displaystyle \sum_{n=0}^\infin a_n y^n 的相关性质。

Abel 定理 对于幂级数 n=0anxn\displaystyle \sum_{n=0}^\infin a_n x^n,有以下结论:

  1. x=x0 (x00)x=x_0\ (x_0\not=0) 是其收敛点,那么当 x<x0|x|<|x_0| 时,n=0anxn\displaystyle \sum_{n=0}^\infin a_n x^n 绝对收敛;
  2. x=x1x=x_1 是其发散点,那么当 x>x1|x|>|x_1| 时,n=0anxn\displaystyle \sum_{n=0}^\infin a_n x^n 发散。

记为:发散点外面也是发散点,收敛点里面也是收敛点。

![image-20230522144148389](C:\Users\Steven Meng\AppData\Roaming\Typora\typora-user-images\image-20230522144148389.png)

推论 幂级数 n=0anxn\displaystyle \sum_{n=0}^\infin a_n x^n 的收敛域仅有以下三种可能情形:

  1. 仅在 x=0x=0 收敛(必然在 x=0x=0 收敛);
  2. 在以原点为中心、长度为 2R(R>0)2R(R>0) 的区间 (R,R)(-R,R) 内绝对收敛,而在 x>R|x|>R 时发散,注:没说 x=R,Rx=R,-R 的情况,可能收敛也可能发散;
  3. (,+)(-\infin,+\infin) 内收敛。

证明:考虑收敛点集的上界。

如果存在一个正数 RR,满足当 x<R|x|<R 时,幂级数 n=1anxn\displaystyle \sum_{n=1}^\infin a_n x^n 绝对收敛;当 x>R|x|>R 时,幂级数 n=1anxn\displaystyle \sum_{n=1}^\infin a_n x^n 发散,则称 RR 为幂级数的 收敛半径。把 (R,R)(-R,R) 称为幂级数的 收敛区间。(和收敛域不同)条件收敛点一定在边界。

R=0R=0 表示仅仅在 x=0x=0 收敛。

![image-20230522145332503](C:\Users\Steven Meng\AppData\Roaming\Typora\typora-user-images\image-20230522145332503.png)

![image-20230524212347710](C:\Users\Steven Meng\AppData\Roaming\Typora\typora-user-images\image-20230524212347710.png)

其中还有一个问题,如何判断 x>R2x>R_2 的敛散性,利用 Abel 定理即可。

接下来,利用比值判别法给出幂级数的收敛半径公式。

系数模比值法

系数模比值法 对于幂级数 n=0anxn\displaystyle \sum_{n=0}^\infin a_n x^n,若:

limnan+1an=ρ+\lim_{n \to \infin} \frac{|a_{n+1}|}{|a_n|}=\rho(或+\infin)

则其收敛半径:

R={0ρ=+,1ρ0<ρ<+,+ρ=0.R=\left\{ \begin{aligned} &0&\rho=+\infin,\\ &\frac{1}{\rho}&0<\rho<+\infin,\\ &+\infin&\rho=0. \end{aligned} \right.

简单理解,需要 x=1/ρx=1/\rho 才能抵消影响。因此为收敛半径。

![image-20230522145047652](C:\Users\Steven Meng\AppData\Roaming\Typora\typora-user-images\image-20230522145047652.png)


做题步骤:

  1. 确定收敛半径 RR.
  2. 讨论端点。

求下列幂级数的收敛域:

n=1(1)nxnn\sum_{n=1}^\infin (-1)^n \frac{x^n}{n}

由于

limnan+1an=limnnn+1=1\lim_{n \to \infin} \frac{|a_{n+1}|}{|a_n|}=\lim_{n \to \infin} \frac{n}{n+1}=1

得到收敛半径为 11,需要 讨论 R=±1\boldsymbol{R=\pm 1}

R=1n=1(1)nnR=1n=11nR=1\Rightarrow \sum_{n=1}^\infin \frac{(-1)^n}{n}\Rightarrow收敛\\ R=-1\Rightarrow \sum_{n=1}^\infin \frac{1}{n} \Rightarrow 发散

因此,收敛域为 (1,1](-1,1]

1+x2!+x24!+x36!+1+\frac{x}{2!}+\frac{x^2}{4!}+\frac{x^3}{6!}+\cdots

limnan+1an=limn1(2(n+1))!1(2n)!=0\lim_{n \to \infin} \frac{|a_{n+1}|}{|a_n|}=\lim_{n \to \infin} \frac{\frac{1}{(2(n+1))!}}{\frac{1}{(2n)!}}=0

因此,收敛域为 (,)(-\infin,\infin)。可以看出来是指数函数的形式罢!

n=12lnnxnn\sum_{n=1}^\infin \frac{2^{\ln n} x^n}{n}

limnan+1an=limn(n+1)ln2nnln2(n+1)=limn(n+1)ln21nln211\lim_{n \to \infin}\frac{|a_{n+1}|}{|a_n|}=\lim_{n \to \infin} \frac{(n+1)^{\ln 2}n}{n^{\ln 2}(n+1)}=\lim_{n \to \infin} \frac{(n+1)^{\ln 2-1}}{n^{\ln 2-1}}\to 1

然后再讨论 R=1,1R=1,-1 的情况。

R=1n=12lnnn=n=1nln21p=ln2+11R=1n=1nln21(1)nR=1\Rightarrow \sum_{n=1}^\infin \frac{2^{\ln n}}{n}=\sum_{n=1}^\infin n^{\ln 2-1}\quad p=-\ln 2+1 \le 1,发散\\ R=-1\Rightarrow \sum_{n=1}^\infin n^{\ln 2-1} (-1)^n 收敛


求幂级数

n=0(x1)n2nn+1\sum_{n=0}^\infin \frac{(x-1)^n}{2^n\sqrt{n+1}}

的收敛域。

首先观察到中心为 11,再计算极限:

n=02nn+12n+1n+2=12\sum_{n=0}^\infin \frac{2^n\sqrt{n+1}}{2^{n+1}\sqrt{n+2}}=\frac{1}{2}

收敛半径为 22,因此,计算

x=3n=01n+1x=1n=0(1)nn+1x=3 \Rightarrow \sum_{n=0}^\infin \frac{1}{\sqrt{n+1}} 发散\\ x=-1\Rightarrow \sum_{n=0}^\infin \frac{(-1)^n}{\sqrt{n+1}} 收敛

因此收敛域为 [1,3)[-1,3).


非标准型幂级数:

n=1(n!)2(2n)!x2n\sum_{n=1}^\infin \frac{(n!)^2}{(2n)!} \cdot x^{2n}

方法1:变量替换;方法2:转化为比值判别法。

![image-20230522145744940](C:\Users\Steven Meng\AppData\Roaming\Typora\typora-user-images\image-20230522145744940.png)

![image-20230522145852121](C:\Users\Steven Meng\AppData\Roaming\Typora\typora-user-images\image-20230522145852121.png)

系数模根值法

系数模根值法 对幂级数 n=0anxn\displaystyle \sum_{n=0}^\infin a_n x^n,若

limnann=ρ(+)\lim_{n \to \infin} \sqrt[n]{|a_n|}=\rho(或+\infin)

则其收敛域半径为:

R={0ρ=+,1ρ0<ρ<+,+ρ=0.R=\left\{ \begin{aligned} &0&\rho=+\infin,\\ &\frac{1}{\rho}&0<\rho<+\infin,\\ &+\infin&\rho=0. \end{aligned} \right.

幂级数的分析性质

连续性定理 若幂级数 n=0anxn\displaystyle \sum_{n=0}^\infin a_n x^n 的收敛半径 R>0R>0,则其和函数 S(x)=n=1anxn\displaystyle S(x)=\sum_{n=1}^\infin a_n x^n(R,R)(-R,R) 内连续;若 n=0anxn\displaystyle \sum_{n=0}^\infin a_n x^nx=Rx=R(或 R-R)收敛,则和函数在 x=Rx=R(或 R-R)左(右)连续。(向收敛区间里连续)

逐项可导性 若幂级数 n=0anxn\displaystyle \sum_{n=0}^\infin a_n x^n 的收敛半径 R>0R>0,则其和函数在 (R,R)(-R,R) 内可导,对于每一项求导,即

S(x)=(n=0anxn)=n=0(anxn)=n=1nanxn1S'(x)=\left(\sum_{n=0}^\infin a_n x^n \right)'=\sum_{n=0}^\infin (a_n x^n )'=\sum_{n=1}^\infin n a_n x^{n-1}

求导所得的幂级数收敛半径仍然为 RR.

如果逐项求导后的级数在 x=Rx=R(或 x=Rx=-R)处收敛,则逐项求导公式在 x=Rx=R(或 x=Rx=-R)处仍然成立。

逐项可积性 类似地,

0xS(t)dt=0xn=0antndt=n=00xantndt=n=0ann+1xn+1\int_0^x S(t)\mathrm{d} t=\int_0^x \sum_{n=0}^\infin a_n t^n \mathrm d t=\sum_{n=0}^\infin \int_0^x a_n t^n \mathrm d t=\sum_{n=0}^\infin \frac{a_n}{n+1}x^{n+1}

这样,幂级数可以看做多项式的推广。

但是,逐项求导,可能会削弱端点收敛性;逐项积分,可能会加强端点的收敛性,但是,均不影响收敛半径。

常用公式

  1. n=0xn=11xx(1,1)\sum_{n=0}^\infin x^n = \frac{1}{1-x} \quad x\in (-1,1)

  2. n=0(x)n=11+xx(1,1)\sum_{n=0}^\infin (-x)^n = \frac{1}{1+x} \quad x\in (-1,1)

  3. n=1nxn1=1(1x)2x(1,1)\boxed{\sum_{n=1}^\infin n x^{n-1} =\frac{1}{(1-x)^2} \quad x\in (-1,1)}

    是上面的求导,也可以看成两项相乘。注意从 11 开始求和。

  4. n=11nxn=ln(1x)x[1,1)\boxed{\sum_{n=1}^\infin \frac{1}{n} x^n =-\ln (1-x) \quad x\in [-1,1)}

    是上面的积分。(收敛半径不变,但收敛域可能改变)

  5. n=1(1)n112n1x2n1=arctanxx[1,1]\sum_{n=1}^\infin (-1)^{n-1} \frac{1}{2n-1} x^{2n-1}=\arctan x \quad x\in[-1,1]

    求导试试。


求下列级数的和:

n=1n(n+1)(1+r)n(r>0)\sum_{n=1}^\infin \frac{n(n+1)}{(1+r)^n} \quad (r>0)

积分一次,得到

n=1n(n+1)(1+r)n+11n+1=n=1n(1+r)n+1\sum_{n=1}^\infin \frac{n(n+1)}{(1+r)^{n+1}}\cdot \frac{-1}{n+1}=\sum_{n=1}^\infin \frac{-n}{(1+r)^{n+1}}


![image-20230522151234488](C:\Users\Steven Meng\AppData\Roaming\Typora\typora-user-images\image-20230522151234488.png)

![image-20230522151315325](C:\Users\Steven Meng\AppData\Roaming\Typora\typora-user-images\image-20230522151315325.png)

先求导,后积分,需要考虑常数。常数由代入特殊值求的。


![image-20230522151549396](C:\Users\Steven Meng\AppData\Roaming\Typora\typora-user-images\image-20230522151549396.png)

![image-20230522151603352](C:\Users\Steven Meng\AppData\Roaming\Typora\typora-user-images\image-20230522151603352.png)

![image-20230522152119953](C:\Users\Steven Meng\AppData\Roaming\Typora\typora-user-images\image-20230522152119953.png)

![image-20230522152132325](C:\Users\Steven Meng\AppData\Roaming\Typora\typora-user-images\image-20230522152132325.png)

先积分,再求导,不用考虑常数。

函数展开为幂级数

回顾一元函数的 nn 阶泰勒公式:

f(x)=f(x0)+f(x0)(xx0)+12!f(x0)(xx0)2++1n!f(n)(x0)(xx0)n+Rn(x)f(x)=f(x_0)+f'(x_0)(x-x_0)+\frac{1}{2!} f''(x_0)(x-x_0)^2+\cdots+\frac{1}{n!} f^{(n)}(x_0)(x-x_0)^n+R_n(x)

其中 Rn(x)=1(n+1)!f(n+1)(ξ)(xx0)n+1\displaystyle R_n(x)=\frac{1}{(n+1)!} f^{(n+1)}(\xi)(x-x_0)^{n+1} 称为 Lagrange 余项,而 Rn(x)=o((xx0)n)\displaystyle R_n(x)=o\left((x-x_0)^n\right) 称为 Peano 余项。当 x0=0x_0=0 时称为麦克劳林公式。

用幂级数的和函数表示 f(x)f(x):

f(x)=a0+a1x+a2x2++anxn+f(x)=a_0+a_1 x+a_2 x^2+\cdots+a_n x^n +\cdots

满足的条件:f(0)=a0f(0)=a_0……

如果 f(x)f(x) 可导,则

f(x)=a1+2a2x+3a3x2+,a1=f(0)f'(x)=a_1+2a_2 x+3a_3 x^2+\cdots ,a_1=f'(0)

f(x)=2a2+6a3x+,a2=12!f(0)f''(x)=2a_2+6a_3 x+\cdots,a_2=\frac{1}{2!} f''(0)

如果函数 f(x)f(x) 的任意阶导数存在,则 an=1n!f(n)(0)\displaystyle a_n=\frac{1}{n!} f^{(n)}(0),故

f(x)=f(0)+f(0)x+12!f(0)x2++1n!f(n)(0)xn+f(x)=f(0)+f'(0)x+\frac{1}{2!} f''(0)x^2+\cdots+\frac{1}{n!} f^{(n)}(0)x^n+\cdots

展开系数是唯一的。

泰勒级数和麦克劳林级数

n=0f(n)(x0)n!(xx0)nx0=0n=0f(n)(0)n!xn\sum_{n=0}^\infin \frac{f^{(n)}(x_0)}{n!} (x-x_0)^n \overset{x_0=0}{\Rightarrow}\sum_{n=0}^\infin \frac{f^{(n)}(0)}{n!} x^n

定理 泰勒级数 n=0f(n)(x0)n!(xx0)n\displaystyle \sum_{n=0}^\infin \frac{f^{(n)}(x_0)}{n!} (x-x_0)^n 在点 x0x_0 的某个邻域内收敛于 f(x)f(x) 的充分必要条件是:函数 f(x)f(x) 在点 x0x_0 处的泰勒公式中余项 Rn(x)R_n(x) 满足 limnRn(x)=0\displaystyle \lim_{n \to \infin} R_n(x)=0


f(x)=exf(x)=e^x 展开为泰勒级数。

证明余项 Rn(x)=eθx(n+1)!xn+1,(0<θ<1)\displaystyle R_n(x)=\frac{e^{\theta x}}{(n+1)!} x^{n+1},(0<\theta<1)nn\to \infin 时趋于 0。

由于幂级数 n=11(n+1)!xn+1\displaystyle \sum_{n=1}^\infin \frac{1}{(n+1)!} x^{n+1} 的收敛域为 (,)(-\infin,\infin),所以 xn+1n+10\displaystyle \frac{x^{n+1}}{n+1}\to 0. 放缩到乘以常数 exe^{x} 也趋于 0。


展开 sinx\sin x.

sinx=x13!x3++sinnπ2n!xn+Rn(x)sinx=xx33!+x55!++(1)n1x2n1(2n1)!+\sin x=x-\frac{1}{3!} x^3+\cdots+\frac{\sin \frac{n\pi}{2}}{n!} x^n+R_n(x)\\ \sin x=x-\frac{x^3}{3!}+\frac{x^5}{5!}+\cdots+(-1)^{n-1} \frac{x^{2n-1}}{(2n-1)!}+\cdots


展开函数 f(x)=(1+x)αf(x)=(1+x)^\alphaxx 的幂级数。

(1+x)α=1+αx+α(α1)2!x2++α(α1)(αn+1)n!xn+(1<x<1)(1+x)^\alpha=1+\alpha x+\frac{\alpha(\alpha-1)}{2!} x^2+\cdots +\frac{\alpha(\alpha-1)\cdots(\alpha-n+1)}{n!} x^n +\cdots (-1<x<1)


11x=1+x+x2++xn+(1<x<1)\frac{1}{1-x} =1+x+x^2+\cdots+x^n+\cdots (-1<x<1)

cosx=(sinx)=1x22!+x44!++(1)nx2n(2n)!+(xR)11+x=1x+x2+(1)nxn+,(1<x<1)ln(1+x)=xx22+x33++(1)n1xnn+,(1<x1)\cos x=(\sin x)'=1-\frac{x^2}{2!}+\frac{x^4}{4!}+\cdots+(-1)^n \frac{x^{2n}}{(2n)!}+\cdots (x\in \R)\\ \frac{1}{1+x}=1-x+x^2-\cdots+(-1)^n x^n +\cdots,{\color{red}(-1<x<1)}\\ \ln(1+x)=x-\frac{x^2}{2}+\frac{x^3}{3}+\cdots+(-1)^{n-1}\frac{x^n}{n}+\cdots,{\color{red}(-1<x\le 1)}


![image-20230524104402154](C:\Users\Steven Meng\AppData\Roaming\Typora\typora-user-images\image-20230524104402154.png)

![image-20230524110154857](C:\Users\Steven Meng\AppData\Roaming\Typora\typora-user-images\image-20230524110154857.png)

注意写展开域在什么范围之内。


将函数 f(x)=arctan12x1+2x\displaystyle f(x)=\arctan \frac{1-2x}{1+2x} 展开为 xx 的幂级数,并求级数 n=1(1)n2n+1\displaystyle \sum_{n=1}^\infin \frac{(-1)^n}{2n+1} 的和。

f(x)=21+4x2=2n=0(1)n(4x2)n=2n=1(1)n4nx2nf'(x)=-\frac{2}{1+4x^2}=-2\sum_{n=0}^\infin (-1)^n (4x^2) ^n=-2\sum_{n=1}^\infin (-1)^n \cdot 4^n \cdot x^{2n}

展开域:x<12|x|<\frac{1}{2}.

积分得:

f(x)=2n=0(4)nx2n+12n+1+C,C=π4f(x)=-2\sum_{n=0}^\infin (-4)^{n} \frac{x^{2n+1}}{2n+1}+C,C=\frac{\pi}{4}

代入 f(12)f(\frac{1}{2}) 即可得到答案。

利用公式 n=1(1)n2n+1x2n+1=arctanx\displaystyle \sum_{n=1}^\infin \frac{(-1)^n}{2n+1} x^{2n+1}=\arctan x 可以直接得到 n=1(1)n2n+1=π4\displaystyle \sum_{n=1}^\infin \frac{(-1)^n}{2n+1}=\frac{\pi}{4}.


幂级数的两大重点内容:展开、求和。常用方法:逐项求导、逐项积分、注意常数。


将函数 f(x)=arctanxf(x)=\arctan x 展开为 xx 的幂级数,并求 f(n)(0)f^{(n)}(0).

利用泰勒级数展开式的唯一性,求 nn 阶导数:

arctanx=n=0(1)n2n+1x2n+1=n=0f(n)n!xn\arctan x = \sum_{n=0}^\infin \frac{(-1)^n}{2n+1} \cdot x^{2n+1}=\sum_{n=0}^\infin \frac{f^{(n)}}{n!} x^n

收敛域:[1,1][-1,1].

f(2n)(0)=0,f(2n+1)(0)=(1)n(2n)!f^{(2n)}(0)=0,f^{(2n+1)}(0)=(-1)^n (2n)!


![image-20230524221058000](C:\Users\Steven Meng\AppData\Roaming\Typora\typora-user-images\image-20230524221058000.png)


image-20230524112931209


展开 f(x)=arctan2+x2x\displaystyle f(x)=\arctan \frac{2+x}{2-x}. arctan\arctan 除了用公式,还可以求导。


总结:

幂级数求和:

  • 阶乘相关函数 利用 xnn!,nxnn!=xxn1(n1)!,n(n1)xnn!=x2xn2(n2)!\displaystyle \frac{x^n}{n!},\frac{nx^n}{n!}=x\cdot \frac{x^{n-1}}{(n-1)!},\frac{n(n-1)x^n}{n!}=x^2 \cdot \frac{x^{n-2}}{(n-2)!}\cdots.
  • 多项式相关函数 利用 nxn=x(xn),n(n1)xn=x2(xn),nx^n=x (x^n)',n(n-1)x^n =x^2 (x^n)'',\cdots

证明题

2

an0a_n \ge 0,级数 n=1an\displaystyle \sum_{n=1}^\infin a_n 收敛,证明:n=1an2\displaystyle \sum_{n=1}^\infin a_n^2 收敛;反之,n=1an2\displaystyle \sum_{n=1}^\infin a_n^2 收敛,n=1an\displaystyle \sum_{n=1}^\infin a_n 不一定收敛。

反例比较好举出,an=1/na_n=1/n。正向证明,使用比值判别法:

limnan2an=0\lim_{n\to \infin} \frac{a^2_n}{a_n}=0

因此 n=1an2\displaystyle \sum_{n=1}^\infin a_n^2收敛。

3

设有二正项级数 n=1an\displaystyle \sum_{n=1}^\infin a_nn=1bn\displaystyle \sum_{n=1}^\infin b_n,其中 an+1anbn+1bn\dfrac{a_{n+1}}{a_n}\le \dfrac{b_{n+1}}{b_n},试证:

(1) 若 n=1bn\displaystyle \sum_{n=1}^\infin b_n 收敛,则 n=1an\displaystyle \sum_{n=1}^\infin a_n 收敛。

关键是利用正项级数的条件,变换题目条件的形式为:

an+1bn+1anbn\frac{a_{n+1}}{b_{n+1}}\le \frac{a_n}{b_n}

因此可以链式不等式:

an+1bn+1anbnan1bn1a1b1\frac{a_{n+1}}{b_{n+1}}\le\frac{a_n}{b_n}\le \frac{a_{n-1}}{b_{n-1}} \le \cdots \le \frac{a_1}{b_1}

因此,使用比较审敛法的极限形式,单调有界必有极限。

limnanbn=A\lim_{n\to \infin} \frac{a_n}{b_n}=A

A0A\ge 0,则 ana_nbnb_n 敛散性相同。

(2) 若 n=1an\displaystyle \sum_{n=1}^\infin a_n 发散,则 n=1bn\displaystyle \sum_{n=1}^\infin b_n 发散。

利用 A0A\ge0 的条件,能得到相同的结论。

4

an0a_n \ge0,而且数列 {nan}\{n a_n\} 有界(也可以表述为 {nan}\{na_n\} 收敛),证明:n=1an2\displaystyle \sum_{n=1}^\infin a_n^2 收敛。

有界的条件:

0nanA0anAn0\le na_n \le A \Rightarrow 0 \le a_n \le \frac{A}{n}

因此

0an2A2n20\le a_n^2 \le \frac{A^2}{n^2}

n=1an2\displaystyle \sum_{n=1}^\infin a_n^2 收敛。

5

an0,bn0a_n \ge 0,b_n \ge 0,而且 n=1an\displaystyle \sum_{n=1}^\infin a_nn=1bn\displaystyle \sum_{n=1}^\infin b_n 都收敛,证明 n=1anbn\displaystyle \sum_{n=1}^\infin a_nb_n 收敛,n=1(an+bn)2\displaystyle \sum_{n=1}^\infin (a_n+b_n)^2 也收敛。

证明简单,只需要观察到 n=1an2\displaystyle \sum_{n=1}^\infin a^2_nn=1bn2\displaystyle \sum_{n=1}^\infin b^2_n 都收敛。

6

an0a_n \ge 0,且 n=1an\displaystyle \sum_{n=1}^\infin a_n 收敛,证明:n=1an/n\displaystyle \sum_{n=1}^\infin a_n/n 收敛。

因为 ann=1nan12(1n2+an2)\displaystyle \frac{a_n}{n}=\frac{1}{n} \cdot a_n\le \frac{1}{2} \left(\frac{1}{n^2}+a_n^2\right)(比较重要的技巧),所以收敛。

7

设数列 {nan}\{na_n\} 收敛,且级数 n=1n(anan1)\displaystyle \sum_{n=1}^\infin n(a_n-a_{n-1}) 收敛,证明:级数 n=1an\displaystyle \sum_{n=1}^\infin a_n 收敛。

只需要打开即可。利用级数收敛相当于部分和收敛。

练习题

11-2

![image-20230521085426378](C:\Users\Steven Meng\AppData\Roaming\Typora\typora-user-images\image-20230521085426378.png)

SN=n=1Nun,σN=n=1N(u2k1+u2k)\displaystyle S_N=\sum_{n=1}^N u_n,\sigma_N=\sum_{n=1}^N (u_{2k-1}+u_{2k})

σN=S2N\sigma_N=S_{2N},只需要证明 S2N+1S_{2N+1}σN\sigma_N 收敛到相同的值。

limnS2n1=limn(S2nu2n)=σ\lim_{n \to \infin} S_{2n-1}=\lim_{n \to \infin}(S_{2n}-u_{2n})=\sigma

因此 limnSn=σ\displaystyle \lim_{n \to \infin} S_n= \sigma

11-3

![image-20230521085944307](C:\Users\Steven Meng\AppData\Roaming\Typora\typora-user-images\image-20230521085944307.png)

看上去像是夹逼定理。但是并不是在问敛散性,而是求值……

这样就要使用万能的 Euler 公式:

Im(qneniα)=qnsinnα\operatorname{Im}(q^ne^{ni\alpha})=q^n \sin n\alpha

则上式为:

qeiα+q2e2iα+q3e3iα++qneniα=qeiα1qeiαqe^{i\alpha}+q^2e^{2i\alpha}+q^3e^{3i\alpha}+\cdots +q^n e^{ni\alpha}=\frac{qe^{i\alpha}}{1-qe^{i\alpha}}

代入展开虚部即可得到:

σ=qcosαq212qcosα+q2\sigma=\frac{q\cos\alpha-q^2}{1-2q\cos\alpha+q^2}

11-4

![image-20230521090944866](C:\Users\Steven Meng\AppData\Roaming\Typora\typora-user-images\image-20230521090944866.png)

还是观察到放缩 1+x11+x \to 1 会比较好。

0<01nx1+xdx<01nxdx=23n3/20 < \int_0^\frac{1}{n} \frac{\sqrt{x}}{1+x}\mathrm d x <\int_0^\frac{1}{n} \sqrt{x}\mathrm{d} x =\frac{2}{3n^{3/2}}

因此收敛。

11-5

判断敛散性:

n=1bn1+an(a>0,b>0)\sum_{n=1}^\infin \frac{b^n}{1+a^n} \quad (a>0,b>0)

使用根值法比较好:

limnunn=limnb1n+ann\lim_{n \to \infin} \sqrt[n]{u_n}= \lim_{n\to\infin} \frac{b}{\sqrt[n]{1^n+a^n}}

0<a<10<a<111 占上风,得到极限是 bb0<b<10<b<1 收敛,b=1b=1 发散,b1b\ge 1 发散。

a1a\ge 1aa 占上风,得到极限是 b/ab/a0<b<a0<b<a 收敛,b=ab=a 发散,bab\ge a 发散。

11-6

![image-20230521092442364](C:\Users\Steven Meng\AppData\Roaming\Typora\typora-user-images\image-20230521092442364.png)

观察到 2=2cosπ4\displaystyle \sqrt{2}=2\cos \frac{\pi}{4},所以:

22=22cosπ4=22sin2π8=2sinπ23\sqrt{2-\sqrt{2}}=\sqrt{2-2\cos \frac{\pi}{4}}=\sqrt{2\cdot 2 \sin^2 \frac{\pi}{8}}=2\sin\frac{\pi}{2^3}

依次递推可以得到

2sinπ22+2sinπ23++2sinπ2n+1+2\sin\frac{\pi}{2^2}+2\sin\frac{\pi}{2^3}+\cdots+2\sin\frac{\pi}{2^{n+1}}+\cdots

因此收敛。

![image-20230521093157029](C:\Users\Steven Meng\AppData\Roaming\Typora\typora-user-images\image-20230521093157029.png)

放缩成更小的 1/(nlnn)1/(n\ln n).

11-7

![image-20230521093816029](C:\Users\Steven Meng\AppData\Roaming\Typora\typora-user-images\image-20230521093816029.png)

第一问利用到

an+an+2=0π4tannxdtanx=1n+1a_n+a_{n+2}=\int_0^\frac{\pi}{4} \tan ^n x \mathrm d \tan x=\frac{1}{n+1}

第二问也是利用这个结论放缩得到

an<1n+1an1nλ+1a_n<\frac{1}{n+1} \Rightarrow a_n \sim\frac{1}{n^{\lambda+1}}

11-8

image-20230521093945944

好像是很古老的题型,利用单调有界必有极限罢!

观察到极限应该是 11

第二题也比较简单,观察到

anan+1an+1ln(1+anan+1an+1)ln(anan+1)\frac{a_n-a_{n+1}}{a_{n+1}}\sim \ln \left(1+\frac{a_n-a_{n+1}}{a_{n+1}}\right)\sim \ln \left(\frac{a_n}{a_{n+1}}\right)

后面的级数应该收敛于 ln2\ln 2. 因此原级数收敛。

11-9

image-20230521094642459

题目条件就是

an1nAa_n \sim\frac{1}{n^A}

![image-20230521094903658](C:\Users\Steven Meng\AppData\Roaming\Typora\typora-user-images\image-20230521094903658.png)

11-10

![image-20230521095038575](C:\Users\Steven Meng\AppData\Roaming\Typora\typora-user-images\image-20230521095038575.png)

![image-20230521095059168](C:\Users\Steven Meng\AppData\Roaming\Typora\typora-user-images\image-20230521095059168.png)

回顾 Leibniz 判别法的三个条件。

11-11

![image-20230521100333093](C:\Users\Steven Meng\AppData\Roaming\Typora\typora-user-images\image-20230521100333093.png)

11-12

研究下列级数的敛散性:

(1) n=1(1)nnan2+3\displaystyle \sum_{n=1}^\infin (-1)^n \frac{n^a}{n^2+3}

还是关注 Lebniz 判别法的三个条件,第一 an>0a_n>0 显然满足,第二点 an0a_n \to 0,当 a2a\ge 2 时显然不满足。而且可以观察到此时是发散的。第三点 an+1<ana_{n+1}<a_n,可以通过求导看:

(nan2+3)=1n2+3na1(3a+(a2)n2)\left(\frac{n^a}{n^2+3}\right)'=\frac{1}{n^2+3}n^{a-1}(3a+(a-2)n^2)

因此 a<2a<2nn 足够大时应该是单调递减的。收敛。

还需要讨论是绝对收敛还是条件收敛

1a<21\le a <2 时,因为 limnun1n2a=limnn2n2+3=1\displaystyle \lim_{n \to \infin}\frac{u_n}{\frac{1}{n^{2-a}}}=\lim_{n\to\infin}\frac{n^2}{n^2+3}=1,所以非绝对收敛。条件收敛。

a<1a<1 时,可得绝对收敛。

(2) n=1(1)nln(2+1n)(3n2)(3n+2)\displaystyle \sum_{n=1}^\infin (-1)^n \frac{\ln \left(2+\frac{1}{n}\right)}{\sqrt{(3n-2)(3n+2)}}

非绝对收敛。条件收敛。

11-13

![image-20230521103108811](C:\Users\Steven Meng\AppData\Roaming\Typora\typora-user-images\image-20230521103108811.png)

(熟悉 Leibniz 判别法的三个条件)首先,ana_n 已经满足单调减少、正项的条件,但是 n=1(1)nan\displaystyle \sum_{n=1}^\infin (-1)^n a_n 发散,这就说明 limnan0\displaystyle \lim_{n \to \infin} a_n \not=0,而且由于 ana_n 单调有界,极限存在。所以 limnan=A>0\displaystyle \lim_{n \to \infin} a_n =A>0.

再证明 $\displaystyle \sum_{n=1}^\infin \left(\frac{1}{a_n+1}\right)^n $ 收敛,使用根值审敛法,1an+11A+1<1\displaystyle \frac{1}{a_n+1} \to \frac{1}{A+1} <1,因此收敛。

11-14

![image-20230521161009596](C:\Users\Steven Meng\AppData\Roaming\Typora\typora-user-images\image-20230521161009596.png)

相当于证明 n=1ann2+λ\displaystyle \sum_{n=1}^\infin \frac{|a_n|}{\sqrt{n^2+\lambda}} 收敛。然后是经典放缩操作:

ann2+λ12(an2+1n2+λ)\frac{|a_n|}{\sqrt{n^2+\lambda}} \le \frac{1}{2}\left(a_n^2+\frac{1}{n^2+\lambda}\right)

因此绝对收敛。

11-15

![image-20230521161210884](C:\Users\Steven Meng\AppData\Roaming\Typora\typora-user-images\image-20230521161210884.png)

(1)

通过比值审敛法:

anan1an1an214an12+an2212\left|\frac{a_n-a_{n-1}}{a_{n-1}-a_{n-2}}\right| \le \frac{1}{4} |a_{n-1}^2+a_{n-2}^2| \le \frac{1}{2}

因此绝对收敛。

(2) 展开 (1) 就好.

考题

11-4

![image-20230523103550344](C:\Users\Steven Meng\AppData\Roaming\Typora\typora-user-images\image-20230523103550344.png)

条件收敛一定在边界上。其他点只可能是绝对收敛或者发散。

11-5

![image-20230523104137705](C:\Users\Steven Meng\AppData\Roaming\Typora\typora-user-images\image-20230523104137705.png)

12-3

![image-20230523102724778](C:\Users\Steven Meng\AppData\Roaming\Typora\typora-user-images\image-20230523102724778.png)

12-5

![image-20230521183805207](C:\Users\Steven Meng\AppData\Roaming\Typora\typora-user-images\image-20230521183805207.png)

(1) 是正确,(2) 是比较经典的做法:

21nun1n2+un22\frac{1}{n} u_n \le \frac{1}{n^2}+u_n^2

因此绝对收敛。

(3) limnun+1un=0\displaystyle \lim_{n \to \infin} \left|\frac{u_{n+1}}{u_n}\right|=0 因此绝对收敛。

(4) 错误,没说正项级数,取 un=(1)n1n,vn=1n\displaystyle u_n=(-1)^n \frac{1}{\sqrt{n}},v_n=\frac{1}{n}.

16-5

![image-20230522204754240](C:\Users\Steven Meng\AppData\Roaming\Typora\typora-user-images\image-20230522204754240.png)

(2). 注意证明方法。

![image-20230522204833124](C:\Users\Steven Meng\AppData\Roaming\Typora\typora-user-images\image-20230522204833124.png)

(3).

16-16

![image-20230522213524483](C:\Users\Steven Meng\AppData\Roaming\Typora\typora-user-images\image-20230522213524483.png)

a0a\le 0 时,limnun0\displaystyle \lim_{n \to \infin} u_n \not=0,因此发散。

0<a<10<a<1 时,使用积分判别法,012x11(2x)α=12ln(2n1)(2n)a+10\displaystyle \int_0^\infin \frac{1}{2x-1}-\frac{1}{(2x)^\alpha}=\left.\frac{1}{2} \ln(2n-1)-(2n)^{-a+1} \right|_0^\infin,因此发散。

![image-20230522213926139](C:\Users\Steven Meng\AppData\Roaming\Typora\typora-user-images\image-20230522213926139.png)

也可以这么比较。

a=1a=1 时,由 Leibniz 法则,收敛。

a>1a>1 时,由发散+收敛=发散,发散。

17-5

![image-20230516091213280](C:\Users\Steven Meng\AppData\Roaming\Typora\typora-user-images\image-20230516091213280.png)

(1) 错误,比如 an+1/an=1/2a_{n+1}/a_n=1/2nn 为偶数,1/2-1/2nn 为奇数。

(2) 正向:

limnanan/(1+an)=1\lim_{n \to \infin } \frac{|a_n|}{|a_n|/(1+|a_n|)}=1

因此敛散性相同,级数

n=1an1+an\sum_{n=1}^\infin \frac{|a_n|}{1+|a_n|}

收敛。

反向:

an<2an1+an|a_n|<\frac{2|a_n|}{1+|a_n|}

因此收敛。

(3)

limnnan/an=+\lim_{n \to \infin } na_n/a_n=+\infin

因此发散。

17-15

![image-20230522194350473](C:\Users\Steven Meng\AppData\Roaming\Typora\typora-user-images\image-20230522194350473.png)

观察到通项等于 1x21x2n+1\displaystyle \frac{1-x^2}{1-x^{2^{n+1}}}.

x>1|x|>1 时,分母 \to\infin,通项趋于 0,当 x<1|x|<1 时趋于 1x21-x^2,当 x=±1x=\pm 1 时,趋于 1/21/2.

而当 x>1|x|>1 时,un12n\displaystyle u_n \le \frac{1}{2^n}.

因此收敛域:(,1][1,+)(-\infin,-1]\cup [1,+\infin).

17-16

求级数 n=1(1)n1(2n+1)3n\displaystyle \sum_{n=1}^\infin \frac{(-1)^{n-1}(2n+1)}{3^n} 的和。

转化为 S(x)=n=1xn(2n+1)=2xn=1nxn1+n=0xn1=3xx2(1x)2\displaystyle S(x)=\sum_{n=1}^\infin x^n (2n+1)=2x \sum_{n=1}^\infin nx^{n-1}+\sum_{n=0}^\infin x^n -1=\frac{3x-x^2}{(1-x)^2}.

得到 S(13)=58\displaystyle -S\left(-\frac{1}{3}\right)=\frac{5}{8}.

17-18

{an}\{a_n\}正项级数Sn=k=1nak\displaystyle S_n=\sum_{k=1}^n a_k

证明:级数 n=1an\displaystyle \sum_{n=1}^\infin a_n 收敛的充要条件是:级数 n=2(1Sn1Sn)\displaystyle \sum_{n=2}^\infin \left(1-\frac{S_{n-1}}{S_n}\right) 收敛。

1SnanSn=anSn1-\frac{S_n-a_n}{S_n}=\frac{a_n}{S_n}

先证明,若级数 n=1an\boldsymbol{\displaystyle \sum_{n=1}^\infin a_n} 收敛,则 n=2anSn\boldsymbol{\displaystyle \sum_{n=2}^\infin \frac{a_n}{S_n}} 收敛。

n=1an=A>0\displaystyle \sum_{n=1}^\infin a_n=A>0 由比较审敛法可以得:

limnananSn=limnSn=A\lim_{n \to \infin} \frac{a_n}{\frac{a_n}{S_n}}=\lim_{n \to \infin} S_n=A

从而两者敛散性相同,级数 n=2anSn\displaystyle \sum_{n=2}^\infin \frac{a_n}{S_n} 收敛。

再证明,如果 n=2anSn\boldsymbol{\displaystyle \sum_{n=2}^\infin \frac{a_n}{S_n}} 收敛,则 n=1an\boldsymbol{\displaystyle \sum_{n=1}^\infin a_n} 收敛。

根据极限收敛的必要条件,得到:

limnSnSn1Sn=0limnSn1Sn=1\lim_{n \to \infin} \frac{S_n-S_{n-1}}{S_n}=0\Rightarrow \lim_{n \to \infin} \frac{S_{n-1}}{S_n}=1

根据 limnSnSn1=1\displaystyle \lim_{n \to \infin} \frac{S_n}{S_{n-1}}=1 知道:

anSn=1Sn1Sn=SnSn1SnSnSn1Sn1lnSnSn1\frac{a_n}{S_n}=1-\frac{S_{n-1}}{S_n}=\frac{S_n-S_{n-1}}{S_n}\sim \frac{S_n-S_{n-1}}{S_{n-1}} \sim \ln \frac{S_n}{S_{n-1}}

从而两者为等价无穷小,假设收敛于 MM,得到

n=2lnSnSn1=limn(lnSnlnS1)=M\sum_{n=2}^\infin \ln \frac{S_n}{S_{n-1}}=\lim_{n\to\infin} (\ln S_n-\ln S_1)=M

因此 limnSn=a1eM\displaystyle \lim_{n \to \infin} S_n= a_1 \mathrm e^M,从而 n=1an\displaystyle \sum_{n=1}^\infin a_n 收敛。


后面是我一开始写的证明。

先证明正向:设 n=1an\displaystyle \sum_{n=1}^\infin a_n 收敛于 A>0A>0,则存在 NN,使得任意 m>Nm>N

A2Sm=n=1man3A2\frac{A}{2}\le S_m=\sum_{n=1}^m a_n \le \frac{3A}{2}

因此,

n=m+1an/(3A/2)n=m+1an/Snn=m+1an/(A/2)\sum_{n=m+1}^\infin a_n/(3A/2)\le \sum_{n=m+1}^\infin a_n/S_n\le \sum_{n=m+1}^\infin a_n/(A/2)

因此收敛。

再证明反向。n=2(1Sn1Sn)\displaystyle \sum_{n=2}^\infin \left(1-\frac{S_{n-1}}{S_n}\right) 收敛也就是

limnSn1Sn=1\lim_{n \to \infin} \frac{S_{n-1}}{S_n}=1

取对数:

n=2lnSn1Sn\sum_{n=2}^\infin \ln \frac{S_{n-1}}{S_n}

因此得到 $S_n \to $ 确定值。

18-4

![image-20230512190658699](C:\Users\Steven Meng\AppData\Roaming\Typora\typora-user-images\image-20230512190658699.png)

A. nπ2n+1\displaystyle \sim n\frac{\pi}{2^{n+1}},由比值判别法这个级数收敛,因此收敛

B. 裂项得到

1n1n+1\frac{1}{\sqrt{n}}-\frac{1}{\sqrt{n+1}}

因此相减得到 11n+111-\frac{1}{\sqrt{n+1}} \to 1,收敛。

C. 还是裂项,显然发散。

D. 凑 Leibniz 判别法

an+1an=n+1(n+1)n+1/nn=nn(n+1)n<1\frac{a_{n+1}}{a_n}=\frac{n+1}{(n+1)^{n+1}/n^n}=\frac{n^n}{(n+1)^n}<1

an0a_n \to 0(利用 n!enn!\sim e^n

因此收敛。

18-5

![image-20230512191531173](C:\Users\Steven Meng\AppData\Roaming\Typora\typora-user-images\image-20230512191531173.png)

  1. bn=anb_n=-a_n,即满足 Leibniz 判别法的条件。

  2. 显然不是这样的罢,比如凑 an=(1)n11na_n=(-1)^{n-1}\frac{1}{n} 就可以证伪。

  3. ![image-20230512193731051](C:\Users\Steven Meng\AppData\Roaming\Typora\typora-user-images\image-20230512193731051.png)

    展开写

18-10

![image-20230522174950134](C:\Users\Steven Meng\AppData\Roaming\Typora\typora-user-images\image-20230522174950134.png)

![image-20230522174957867](C:\Users\Steven Meng\AppData\Roaming\Typora\typora-user-images\image-20230522174957867.png)

18-17

![image-20230522173329989](C:\Users\Steven Meng\AppData\Roaming\Typora\typora-user-images\image-20230522173329989.png)

![image-20230522173803071](C:\Users\Steven Meng\AppData\Roaming\Typora\typora-user-images\image-20230522173803071.png)

18-18

![image-20230522164819689](C:\Users\Steven Meng\AppData\Roaming\Typora\typora-user-images\image-20230522164819689.png)

(1)

首先收敛半径的定义:

对于任意 x(1,1)x\in (-1,1),幂级数绝对收敛,因为 ana_n 是正项级数,也就是对于任意 x[0,1)x\in [0,1),幂级数 n=1anxn\displaystyle \sum_{n=1}^\infin a_n x^n 收敛。由数列有界的定义,M>0,n=1,2,3,,0<an<M\exists M>0,\forall n=1,2,3,\cdots, 0<a_n <M。那么构造收敛级数 n=1Mxn\displaystyle \sum_{n=1}^\infin M x^n,由比较审敛法可以知道级数收敛。

再证明对于任意 x>1|x|>1,当 x>1x>1,构造发散级数 n=1an\displaystyle \sum_{n=1}^\infin a_n,由比较审敛法可以知道级数发散。由 Abel 定理,x<1x<-1 也是发散的。

(2)

条件是无法直接导出数列 {an}\{a_n\} 有界的。比如取 an=na_n=n……

但是其实就是 limnanSn=0\displaystyle \lim_{n \to \infin} \frac{a_n}{S_n}=0 这个东西非常经典。

也就是 ε,N,n>N,ε<anSn<ε\forall \varepsilon,\displaystyle \exists N,\forall n>N,-\varepsilon < \frac{a_n}{S_n}<\varepsilon.

εSn+1<an+1<εSn+1-\varepsilon S_{n+1}<a_{n+1}<\varepsilon S_{n+1}

εSn<an<εSn-\varepsilon S_n <a_n <\varepsilon S_n.

得到

εan+1<an+1an<εan+1-\varepsilon a_{n+1}<a_{n+1}-a_{n}<\varepsilon a_{n+1}

ε<1anan+1<ε\displaystyle -\varepsilon <1-\frac{a_n}{a_{n+1}}<\varepsilon

也就是 limnan+1an=1\displaystyle \lim_{n \to \infin} \frac{|a_{n+1}|}{|a_n|}=1.

![image-20230522172118196](C:\Users\Steven Meng\AppData\Roaming\Typora\typora-user-images\image-20230522172118196.png)

19-10![image-20230521212911353](C:\Users\Steven Meng\AppData\Roaming\Typora\typora-user-images\image-20230521212911353.png)

19-16

![image-20230521213715109](C:\Users\Steven Meng\AppData\Roaming\Typora\typora-user-images\image-20230521213715109.png)

19-24

![image-20230521215920836](C:\Users\Steven Meng\AppData\Roaming\Typora\typora-user-images\image-20230521215920836.png)

19-28

![image-20230521224321563](C:\Users\Steven Meng\AppData\Roaming\Typora\typora-user-images\image-20230521224321563.png)

19-30

![image-20230521230501417](C:\Users\Steven Meng\AppData\Roaming\Typora\typora-user-images\image-20230521230501417.png)

![image-20230521231131440](C:\Users\Steven Meng\AppData\Roaming\Typora\typora-user-images\image-20230521231131440.png)

19-39

![image-20230517103037808](C:\Users\Steven Meng\AppData\Roaming\Typora\typora-user-images\image-20230517103037808.png)

(3).

(4). 是对的,取 bn=nanb_n=na_n,因此 limnbnan=+\displaystyle \lim_{n \to \infin }\frac{b_n}{a_n}=+\infin,是发散的。

20-4

![image-20230512185644980](C:\Users\Steven Meng\AppData\Roaming\Typora\typora-user-images\image-20230512185644980.png)

A. 显然 1n32\sim \frac{1}{n^\frac{3}{2}},收敛

B. (11n)n1e(1-\frac{1}{n})^n \to \frac{1}{e},发散

C. 拆开形成 n+1n(nn1)\sqrt{n+1}-\sqrt{n}-(\sqrt{n}-\sqrt{n-1}),然后可以得到收敛

D. 1cos1n1-\cos \frac{1}{n} 单减,趋于 0,因此收敛

20-5

![image-20230520223417550](C:\Users\Steven Meng\AppData\Roaming\Typora\typora-user-images\image-20230520223417550.png)

(2) 构造反例 an=1,n%2=0;1/n4,n%2=1a_n=1,n\% 2=0;1/n^4 ,n\%2=1.

20-10

![image-20230512185942822](C:\Users\Steven Meng\AppData\Roaming\Typora\typora-user-images\image-20230512185942822.png)

代换 x1/ux \to 1/u,得到

1ln(1+1/u2)1/u1u2du=1ln(1+1/u2)udu\int_1^\infin -\frac{\ln (1+1/u^2)}{1/u} \frac{1}{u^2} \mathrm d u=\int_1^\infin -\frac{\ln (1+1/u^2)}{u}\mathrm d u

因此等于

n=1ln(1+1/n2)n\sum_{n=1}^\infin -\frac{\ln (1+1/n^2)}{n}

21-2

![image-20230517200221850](C:\Users\Steven Meng\AppData\Roaming\Typora\typora-user-images\image-20230517200221850.png)

找等价。

(1) n=1(1)nn3/4\displaystyle \sum_{n=1}^\infin \frac{(-1)^n}{n^{3/4}},条件收敛。

(2) n=1(1)nn3/2\displaystyle \sum_{n=1}^\infin \frac{(-1)^n}{n^{3/2}},加了绝对值之后收敛,不是条件收敛。

(3) n=1(1)n1nπ\displaystyle \sim \sum_{n=1}^\infin (-1)^n \frac{1}{n}\pi,条件收敛。

(4) 第一部分绝对收敛(夹逼定理),第二部分发散,原来就是发散的,不是条件收敛。

21-7

![image-20230517200052441](C:\Users\Steven Meng\AppData\Roaming\Typora\typora-user-images\image-20230517200052441.png)

21-8

![image-20230517202255938](C:\Users\Steven Meng\AppData\Roaming\Typora\typora-user-images\image-20230517202255938.png)

21-15

![image-20230517204931508](C:\Users\Steven Meng\AppData\Roaming\Typora\typora-user-images\image-20230517204931508.png)

21-17

![image-20230517205140164](C:\Users\Steven Meng\AppData\Roaming\Typora\typora-user-images\image-20230517205140164.png)

事两边求交集。

21-21

![image-20230517211005693](C:\Users\Steven Meng\AppData\Roaming\Typora\typora-user-images\image-20230517211005693.png)

21-24

![image-20230521191349560](C:\Users\Steven Meng\AppData\Roaming\Typora\typora-user-images\image-20230521191349560.png)

![image-20230521191410988](C:\Users\Steven Meng\AppData\Roaming\Typora\typora-user-images\image-20230521191410988.png)

21-25

![image-20230517194658844](C:\Users\Steven Meng\AppData\Roaming\Typora\typora-user-images\image-20230517194658844.png)

(1) 条件收敛的点必然位于收敛半径上,从而收敛半径为 1,逐项求导不改变收敛半径,从而命题得证;

对于 (2),考虑反证,如果 n=1(an+bn)\displaystyle \sum_{n=1}^\infin(|a_n|+|b_n|) 收敛,因为都是正项级数,所以两个级数 n=1an,n=1bn\displaystyle \sum_{n=1}^\infin |a_n|,\displaystyle \sum_{n=1}^\infin |b_n| 均收敛!

(3) 只要展开级数,得到 limnun+12u12\lim_{n \to \infin}u_{n+1}^2-u_1^2 收敛,即可。

(4)

n=1nun\displaystyle \sum_{n=1}^\infin n u_n 绝对收敛得到 n=1nun\displaystyle \sum_{n=1}^\infin n |u_n| 收敛,也就是 n=1un\displaystyle \sum_{n=1}^\infin |u_n| 收敛,顺便 n=1un\displaystyle \sum_{n=1}^\infin u_n 收敛。

n=1vn/n\displaystyle \sum_{n=1}^\infin v_n/n 条件收敛得到:

  1. n=1vn/n\displaystyle \sum_{n=1}^\infin v_n/n 收敛,此时 vn\sum v_n 可以发散也可以收敛。
  2. n=1vn/n\displaystyle\sum_{n=1}^\infin |v_n|/n 发散,此时 vn\sum|v_n| 一定发散。

再看看 n=1(un+vn)\displaystyle \sum_{n=1}^\infin (u_n+v_n) 条件收敛的条件,要求 n=1(un+vn)\displaystyle \sum_{n=1}^\infin (u_n+v_n) 收敛,n=1un+vn\displaystyle \sum_{n=1}^\infin |u_n+v_n| 发散。

感觉是有反例的罢,比如取 un=0,vn=(1)nu_n=0,v_n=(-1)^n,就是反例罢!白推了这么多。

21-35

![image-20230517164549432](C:\Users\Steven Meng\AppData\Roaming\Typora\typora-user-images\image-20230517164549432.png)

主要利用到 lnn\ln n 的性质,n,lnnn \to \infin,\ln n \to \infin,但是 ln(n)n0\ln (n) \sim n^0

(1) 考虑到判别条件 limnan/(1/n)=0\lim_{n \to \infin} a_n / (1/n)=0 但是 << 发散并不能说明收敛,比如取 an=1/(nlnn)a_n=1/(n\ln n).

(2) 也是一样的例子,取 an=1/(nlnn)a_n=1/(n\ln n)

(3) 题目前面应该上限是 nn,参见 17-18 的证明方法。

(4) 这种不太好利用比值判别法的,一般都是比较判别法。

利用不等式 ab12(a+b)\displaystyle \sqrt{ab}\le\frac{1}{2}(a+b),可以得到

annp12(an+1n2p)\frac{\sqrt{a_n}}{n^p} \le \frac{1}{2}\left(a_n+\frac{1}{n^{2p}}\right)

为一个收敛级数+一个收敛级数,因此收敛。

(5)

也是框定 1anbn\displaystyle \frac{1}{a^n-b^n} 的范围。需要缩小分母,得到一个比原来数大的值。

1anbn=1(ab)(an1+an2b++bn1)1n(ab)bn1\frac{1}{a^n-b^n}=\frac{1}{(a-b)(a^{n-1}+a^{n-2}b+\cdots+b^{n-1})}\le \frac{1}{n(a-b)b^{n-1}}

因此收敛。

也可以这么做:

1anbn=1bn1(a/b)n1<1bn,n\frac{1}{a^n-b^n}=\frac{1}{b^n}\cdot \frac{1}{(a/b)^n -1} <\frac{1}{b^n},n\to\infin

因此成立。

期末讲座

1

已知函数 f(x)=xln11x\displaystyle f(x)=\frac{x}{\ln \frac{1}{1-x}} 在区间 (0,12)\displaystyle \left(0,\frac{1}{2}\right) 上能展开为幂级数 n=0cnxn\displaystyle \sum_{n=0}^\infin c_n x^n,求 c0c_0.

方法 1:求 c0c_0 不能直接代入 x=0x=0,但是可以求 limx0+f(x)=1\displaystyle \lim_{x\to 0^+} f(x)=1(延拓的思想,还可以进行偶延拓),因此 c0=1c_0=1.

方法 2:求 xx+x22+x33+\displaystyle \frac{x}{x+\frac{x^2}{2}+\frac{x^3}{3}+\cdots}.

2

![image-20230525162145155](C:\Users\Steven Meng\AppData\Roaming\Typora\typora-user-images\image-20230525162145155.png)

没有收敛最慢的收敛级数,也没有发散最快的发散级数,

给出反例:

an={1n2,n1n,na_n=\left\{ \begin{aligned} &\frac{1}{n^2},n不是完全平方数\\ &\frac{1}{n},n是完全平方数\\ \end{aligned} \right.

因此第二个命题是错误的。

第一个命题:例如 an=1lnn\displaystyle a_n=\frac{1}{\ln n},其无穷小阶小于任何正数,但不是 0。

3

求级数 n=0(1)n4n+1\displaystyle \sum_{n=0}^\infin \frac{(-1)^n}{4n+1}

思路一:考虑幂级数 S(x)=n=0(1)n4n+1x4n+1\displaystyle S(x)=\sum_{n=0}^\infin \frac{(-1)^n}{4n+1} x^{4n+1};求导。

思路二:直接凑,熟悉 1p+1=01xpdx\displaystyle \frac{1}{p+1}=\int_0^1 x^p \mathrm{d} x.

![image-20230525163650311](C:\Users\Steven Meng\AppData\Roaming\Typora\typora-user-images\image-20230525163650311.png)

4

已知数列 {an}\{a_n\} 满足 a0=9,a1=4,an2n(n1)an=0(n2)a_0=9,a_1=4,a_{n-2}-n(n-1)a_n=0(n\ge 2),记 S(x)=n=0anxn\displaystyle S(x)=\sum_{n=0}^\infin a_nx^n,求 S(110)\displaystyle S\left(\frac{1}{10}\right)

S(x)=n=2n(n1)anxn2=n=2an2xn2=n=0anxn=S(x)S''(x)=\sum_{n=2}^\infin n(n-1) a_n x^{n-2}=\sum_{n=2}^\infin a_{n-2} x^{n-2}=\sum_{n=0}^\infin a_n x^n =S(x)

再使用初值条件:S(0)=9,S(0)=4S(0)=9,S'(0)=4. 微分方程的解是 (Ax+B)ex(Ax+B)e^x,则 B=9,A=5B=9,A=-5

评论